Exercice 2

Partie

Question

On considère les séries entières suivantes :

\(A. \displaystyle\sum_{n\geq 2}\frac{z^n}{n(n-1)}\) \(B. \sum\frac{3n}{n+2}z^n\) \(C.\sum\frac{n^2+n-1}{n!}z^n\).

Pour chacune d'entre elles on répondra aux questions suivantes :

1. Quel est le rayon de convergence R de la série entière ?

Solution détaillée

A. On a : \(\forall n\geq 2, a_n>0\), et \(\frac{a_{n+1}}{a_n}=\frac{n-1}{n+1}\). On en déduit :

\(\displaystyle\lim_{n\rightarrow +\infty}\frac{a_{n+1}}{a_n}=1\) et donc \(R=1\).

B. On a : \(\forall n\geq 1, a_n >0\), et \(\frac{a_{n+1}}{a_n}=\frac{(n+1)(n+2)}{n(n+3)}\).

D'où \(\displaystyle\lim_{n\rightarrow +\infty}\frac{a_{n+1}}{a_n}=1\) et donc \(R=1\).

C. On a : \(\forall n\in N, a_n>0\). On forme le rapport \(\frac{a_{n+1}}{a_n}=\frac{(n+1)^2+n}{n^2+n-1}.\frac{1}{n+1}\).

On en déduit \(\displaystyle\lim_{n\rightarrow +\infty}\frac{a_{n+1}}{a_n}=0\), et donc \(R=+\infty\).

Question

2. La série est-elle convergente en des points du cercle \(C (0, R)\) ? Si oui, lesquels ? non posée pour B (R est infini)

Solution détaillée

A. Sur le cercle unité, on a : \(\left|\frac{z^n}{n(n-1)}\right|=\frac{1}{n(n-1)}\) et la série de terme général \(\frac{z^n}{n(n-1)}\)est absolument convergente en tout point du cercle.

B. Sur le cercle unité, on a : \(\left|\frac{3nz^n}{(n+2)}\right|=\frac{3n}{(n+2)}\), le terme général de la série ne tend pas vers 0 et la série de terme général \(\frac{3nz^n}{n+2}\)est divergente en tout point du cercle unité.

C. Le problème ne se pose pas puisque le rayon de convergence est infini.

Question

3. On se limite au cas de variable réelle. Calculer la somme de la série entière dans l'intervalle de convergence \(]-R,R[\).

Solution détaillée

A. A l'intérieur de l'intervalle \(]-1,1[\), on peut dériver et intégrer terme à terme la série entière. On pose : \(\forall x\in ]-1,1[\), \(F(x)=\displaystyle\sum_{n=2}^{+\infty}\frac{x^n}{n(n-1)}\). On en déduit : \(F'(x)=\displaystyle\sum_{n=2}^{+\infty}\frac{x^{n-1}}{n-1}=\displaystyle\sum_{k=1}^{+\infty}\frac{x^k}{k}\). Il s'agit (au signe près) d'un développement classique, on a : \(F'(x)=-\ln{(1-x)}\) et donc \(F(x)=F(0)-\int_0^x\ln{(1-t)}dt=x+(1-x)\ln(1-x)\).

B. Pour tout \(x\) appartenant à l'intervalle \(]-1,1[\), on pose : \(G(x)=\displaystyle\sum_{n=0}^{+\infty}\frac{3n}{n+2}x^n\). On peut écrire \(G(x)=\displaystyle\sum_{n=0}^{+\infty}\frac{3(n+2)-6}{n+2}x^n=\displaystyle\sum_{n=0}^{+\infty}3x^n-6\displaystyle\sum_{n=0}^{+\infty}\frac{x^n}{n+2}\) car les deux séries qui interviennent ont même rayon de convergence que la série étudiée. On a, pour tout \(x\) non nul et appartenant à l'intervalle \(]-1,1[\), \(\displaystyle\sum_{n=0}^{+\infty}\frac{x^n}{n+2}=\frac{1}{x^2}\displaystyle\sum_{n=0}^{+\infty}\frac{x^{n+2}}{n+2}=\frac{1}{x^2}\displaystyle\sum_{k=2}^{+\infty}\frac{x^k}{k}=\frac{1}{x^2}\left(\displaystyle\sum_{k=1}^{+\infty}\frac{x^k}{k}-x\right)\). D'où : \(G(x)=\frac{3}{1-x}+6\frac{\ln{(1-x)}+x}{x^2}\).

C. Pour tout \(x\) réel on pose : \(H(x)=\displaystyle\sum_{n=0}^{+\infty}\frac{n2+n-1}{n!}x^n\). En écrivant : \(n^2+n-1=n(n-1)+2n-1\), on obtient : \(H(x)=\displaystyle\sum_{n=2}^{+\infty}\frac{n(n-1)}{n!}x^n+2\displaystyle\sum_{n=1}^{+\infty}\frac{x^n}{(n-1)!}-\displaystyle\sum_{n=0}^{+\infty}\frac{x^n}{n!}\). (Chacune des séries considérées a un rayon de convergence infini, l'opération est donc légitime). On en déduit : \(H(x)=x^2\displaystyle\sum_{n=2}^{+\infty}\frac{x^{n-2}}{(n-2)!}+2x\displaystyle\sum_{n=1}^{+\infty}\frac{x^{n-1}}{(n-1)!}-\displaystyle\sum_{n=0}^{+\infty}\frac{x^n}{n!}\), soit encore, \(H(x)=\left(\displaystyle\sum_{k=0}^{+\infty}\frac{x^k}{k!}\right)(x^2+2x-1)\) et finalement \(H(x)=e^x(x^2+2x-1)\).